ChaseDream

标题: 【求助GWD TN24-17 Q41】 [打印本页]

作者: 水色青空    时间: 2010-3-6 13:23
标题: 【求助GWD TN24-17 Q41】
41.: GWD-23-Q39
In Gandania, where the government has a monopoly on tobacco sales, the incidence of smoking-related health problems has risen steadily for the last twenty years. The health secretary recently proposed a series of laws aimed at curtailing tobacco use in Gandania. Profits from tobacco sales, however, account for ten percent of Gandania’s annual revenues. Therefore, Gandania cannot afford to institute the proposed laws.

Which of the following, if true, most seriously weakens the argument?

A.All health care in Gandania is government-funded.

B.Implementing the proposed laws is not likely to cause a significant increase in the amount of tobacco Gandania exports.

C.The percentage of revenue Gandania receives from tobacco sales has remained steady in recent years.

D.Profits from tobacco sales far surpass any other single source of revenue for the Gandanian government.

E.No government official in Gandania has ever previously proposed laws aimed at curtailing tobacco use.












答案选A~~~想请教B为什么不对~~唯一的可能是不是我把AFFORD理解错了??~~求助~~~

作者: 水色青空    时间: 2010-3-6 13:27
顶~
作者: mycontrol01    时间: 2010-3-6 14:12
B说的是采取措施后出口增加不了     而国内的销售却会减少 意思就是政府收入确实会降低 所以B削弱不了

不知道你怎么想的


我选A了  B我没有取非,就是直接排除的
作者: 水色青空    时间: 2010-3-6 16:04
B说的是采取措施后出口增加不了     而国内的销售却会减少 意思就是政府收入确实会降低 所以B削弱不了

不知道你怎么想的


我选A了  B我没有取非,就是直接排除的
-- by 会员 mycontrol01 (2010/3/6 14:12:21)





明白了~




欢迎光临 ChaseDream (https://forum.chasedream.com/) Powered by Discuz! X3.3